LSAT and Law School Admissions Forum

Get expert LSAT preparation and law school admissions advice from PowerScore Test Preparation.

 rpark8214
  • Posts: 23
  • Joined: Apr 27, 2017
|
#34786
Hi,
I chose answer choice (D) under the impression that this country would be motivated by a subjective view over objective calculations. Because this piece of territory was claimed by country B, country A would "take huge gambles to retrieve what they perceive to have been taken from them" (lines 45-47).

Is answer (D) incorrect because the situation is not one where the risk far outweighs the objectively measurable value of assets, but rather one where there is moderate risk paired with great reward (mineral wealth)? Thanks for your input!
 Adam Tyson
PowerScore Staff
  • PowerScore Staff
  • Posts: 5153
  • Joined: Apr 14, 2011
|
#34916
Thanks for asking, rpark! Here's my problem with answer D - it's too certain. While I might be okay with "the facts of the situation suggest that the government may be motivated by other factors" (such as a perception that the territory in question had been taken from them previously), I cannot say with any confidence that the author would be so sure that that's what happened in this case.

Answer A harkens back to the "previously accepted views" described around lines 16-23. Those previously accepted views are that the government in our example would be willing to accept a moderate, tolerable risk in exchange for a great reward. That's what they did, and that complies with those views. That's what makes A the best choice here - our author would say that the government in question was following the old school pattern and not the "huge gamble" approach.

I hope that helps! Keep at it!
User avatar
 German.Steel
  • Posts: 55
  • Joined: Jun 12, 2021
|
#99124
(A) makes sense, but I'm having a bit of trouble dispatching (C). In this situation, a country is seizing territory from another country - which, based on the new research, implies that the country having its land seized may be inclined to go to extreme lengths/risks in order to get it back. This would suggest that the potential value of the gain may, possibly, be overridden by the risks.

Granted, it takes too many assumptions to connect the dots on making (C) the winner, especially given how strongly (C) is worded ("is overridden by the risks" as opposed to something weaker like "may be overridden by the risks"). But I guess I'm wondering: if you did swap out "is overridden by the risks" with "may be overridden by the risks," would (C) become a solid answer? Or am I overlooking something else that makes (C) unattractive? Is (C) just fighting the facts too much, given that the question says "moderate but easily tolerable harm"? Thanks in advance!!
User avatar
 Jeff Wren
PowerScore Staff
  • PowerScore Staff
  • Posts: 385
  • Joined: Oct 19, 2022
|
#99270
Hi German.Steel,

You're exactly right in guessing that Answer C is "just fighting the facts too much."

The key here is paying attention to the exact wording of the hypothetical facts that are given in the question. By using the phrase "a concomitant risk of failure involving moderate but easily tolerable harm in the long run" the hypothetical is stating this level of risk as a fact in the hypothetical.

If the wording had been changed to something describing the first country's perception or belief that that risk was moderate (but they could be wrong), then that would make Answer C more appealing.

Get the most out of your LSAT Prep Plus subscription.

Analyze and track your performance with our Testing and Analytics Package.